All Subgroups of S3: Lagrange's Theorem Explained

  • Thread starter Thread starter e179285
  • Start date Start date
  • Tags Tags
    Subgroup
Click For Summary
All subgroups of S3 can have orders of 1, 2, 3, or 6, as dictated by Lagrange's Theorem. The subgroup of order 1 is the identity permutation (1). There are three distinct subgroups of order 2: {e, (12)}, {e, (13)}, and {e, (23)}, while the single subgroup of order 3 is {e, (123), (132)}. The entire group S3 serves as the subgroup of order 6. The discussion clarifies that "H5" is not applicable since all subgroup orders must be divisors of 6.
e179285
Messages
24
Reaction score
0
The question wants all subgroups of S3 . If H≤S 3 , then ; IHI=1,2,3,6 by Lagrance's Theorem.

In other words, order of H can be 1,2,3 and 6.

What ı want to ask is how to write subgroup of S3. For example,is H 1 (1) ?
 
Physics news on Phys.org
For "H_1", yes, any subgroup must contain the identity so if H contains only one member, it must be just the identity permutation, (1).

There are, in fact, 3 different subgroups of order 2: {e, (12)}, {e, (13)}, and {e, (23)}. There is the single subgroup of order 3: {e, (123), (132)}. Of course, the subgroup of order 6 is the entire group.
 
HallsofIvy said:
For "H_1", yes, any subgroup must contain the identity so if H contains only one member, it must be just the identity permutation, (1).

There are, in fact, 3 different subgroups of order 2: {e, (12)}, {e, (13)}, and {e, (23)}. There is the single subgroup of order 3: {e, (123), (132)}. Of course, the subgroup of order 6 is the entire group.

What can H5 be?
 
? You just said that every subgroup of S3 (every subgroup of any group of order 6) must have order 1, 2, 3, or 6 (a divisor of 6). What do you mean by "H5"?
 
Question: A clock's minute hand has length 4 and its hour hand has length 3. What is the distance between the tips at the moment when it is increasing most rapidly?(Putnam Exam Question) Answer: Making assumption that both the hands moves at constant angular velocities, the answer is ## \sqrt{7} .## But don't you think this assumption is somewhat doubtful and wrong?

Similar threads

  • · Replies 6 ·
Replies
6
Views
2K
  • · Replies 4 ·
Replies
4
Views
3K
  • · Replies 7 ·
Replies
7
Views
3K
  • · Replies 23 ·
Replies
23
Views
3K
Replies
2
Views
2K
  • · Replies 2 ·
Replies
2
Views
6K
  • · Replies 23 ·
Replies
23
Views
3K
  • · Replies 14 ·
Replies
14
Views
5K
Replies
6
Views
3K
  • · Replies 9 ·
Replies
9
Views
2K